On the divisibility of Fermat quotients (Q1958002)

From MaRDI portal
Revision as of 07:07, 3 July 2024 by ReferenceBot (talk | contribs) (‎Changed an Item)
(diff) ← Older revision | Latest revision (diff) | Newer revision → (diff)
scientific article
Language Label Description Also known as
English
On the divisibility of Fermat quotients
scientific article

    Statements

    On the divisibility of Fermat quotients (English)
    0 references
    0 references
    0 references
    0 references
    0 references
    28 September 2010
    0 references
    For a prime \(p\) and an integer \(a\) the Fermat quotient is defined as \[ q_p(a)=\frac{a^{p-1}-1}{p}. \] Let \(l_p\) be the smallest value of \(a\) for which \(q_p(a)\not\equiv 0\pmod p\). The authors prove that \[ l_p\leq (\log p)^{\frac{463}{252}+o(1)} \] as \(p\to\infty\). As a corollary, for every \(\varepsilon>0\) and a sufficiently large integer \(n\), if \(a^{n-1}\equiv 1\pmod n\) for every positive integer \(a\leq (\log n)^{\frac{463}{252}+\varepsilon}\) then \(n\) is squarefree. Furthermore, for every \(\varepsilon>0\), there is \(\delta>0\) such that for all but one prime \(Q^{1-\delta} <p\leq Q\), we have \(l_p\leq (\log p)^{\frac{59}{35}+\varepsilon}\). Finally, for every \(\varepsilon>0\), there is \(\delta>0\) such that for all but \(O(Q^{1-\delta})\) primes \(p\leq Q\), we have \(l_p\leq (\log p)^{\frac{5}{3}+\varepsilon}\).
    0 references
    0 references
    0 references
    0 references
    0 references
    0 references
    Fermat quotient
    0 references
    0 references